Đến nội dung

tpdtthltvp nội dung

Có 806 mục bởi tpdtthltvp (Tìm giới hạn từ 25-05-2020)



Sắp theo                Sắp xếp  

#654920 $Max P=\dfrac{x}{y}+\dfrac{y}...

Đã gửi bởi tpdtthltvp on 20-09-2016 - 21:11 trong Bất đẳng thức và cực trị

Cho $ x, y, z \in [1;3] $ .

 

Tìm MAX của biểu thức:

 

$$P=\dfrac{x}{y}+\dfrac{y}{z}+\dfrac{z}{x}+\dfrac{y}{x}+\dfrac{x}{z}+\dfrac{z}{y}$$

Do vai trò của $x,y,z$ như nhau nên không mất tính tổng quát, giả sử  $1\leq x\leq y\leq z\leq 3.$

Dễ dàng suy ra: $\frac{x}{z}\leq 3;\frac{z}{x}\leq 3.$ Do đó: $(3-\frac{x}{z})(3-\frac{z}{x})\geq 0\Leftrightarrow \frac{x}{z}+\frac{z}{x}\leq \frac{10}{3}(1)$

Mặt khác ta có: $\frac{x}{y}\leq 1;\frac{y}{z}\leq 1$ nên $(1-\frac{x}{y})(1-\frac{y}{z}) \geq 0 \Leftrightarrow \frac{x}{y}+\frac{y}{z}\leq 1+\frac{x}{z}(2)$

Tương tự, ta được: $\frac{y}{x}+\frac{z}{y}\leq 1+\frac{z}{x}(3)$

Từ $(1),(2),(3)$ suy ra $\frac{x}{y}+\frac{y}{z}+\frac{y}{x}+\frac{z}{y}\leq 2+\frac{x}{z}+\frac{z}{x}\leq 2+\frac{10}{3}=\frac{16}{3}$

Do đó: $P=\dfrac{x}{y}+\dfrac{y}{z}+\dfrac{z}{x}+\dfrac{y}{x}+\dfrac{x}{z}+\dfrac{z}{y} \leq \frac{16}{3}+\frac{10}{3}=\frac{26}{3}$

Dấu $"="$ xảy ra khi $a,b,c$ có $1$ số bằng $1;\;\;\ 2$ số bẳng $3$ hoặc có $1$ số bằng $3; \;\;\ 2$ số bằng $1.$ 




#654208 Tìm $v_{n}$

Đã gửi bởi tpdtthltvp on 14-09-2016 - 21:10 trong Số học

Cho $v_{0}=2;v_{1}=3;v_{k+1}=3v_{k}-2v_{k-1} \forall k\geq 1$. Tìm $v_{n}$

Viết lại công thức truy hồi dưới dạng:    $v_{k+1}-3v_k+2v_{k-1}=0$ suy ra phương tình đặc trưng của dãy có dạng $\lambda ^2-3\lambda +2=0.$ Phương trình này có $2$ nghiệm là $\lambda _1=1,\lambda _2=2,$ vì thế theo lý thuyết dãy số, $v_n$ được biểu diễn dưới dạng: 

$$v_n=\alpha _1(1)^n+\alpha _2(2)^n;n=0,1,...$$

Mà ta có: $v_0=2;v_1=3$ nên:

$$\left\{\begin{matrix} \alpha _1+\alpha _2=2 \\ \alpha _1+2\alpha _2=3 \end{matrix}\right.\Rightarrow \alpha _1=\alpha _2=1$$

Vậy $v_n=1^n+2^n=1+2^n$




#654052 Các chủ đề bị khóa

Đã gửi bởi tpdtthltvp on 13-09-2016 - 20:55 trong Hướng dẫn - Trợ giúp - Giải đáp thắc mắc khi sử dụng Diễn đàn

Do bạn đặt tiêu đề không đúng quy định :)




#653370 Thăm dò ý kiến về việc thi trắc nghiệm môn toán

Đã gửi bởi tpdtthltvp on 08-09-2016 - 20:08 trong Tin tức - Vấn đề - Sự kiện

"Tạm biệt phương trình thân yêu
 Tạm biệt Oxy luôn nhé
 Tạm biệt bất đẳng thức xinh xinh
 Mai em thi trắc nghiệm rồi
 Nhớ lắm quên sao được
 Bộ giáo dục thân yêu" 
                         (Nguồn: Facebook)
:D :D



#652860 Inequalities From 2016 Mathematical Olympiads

Đã gửi bởi tpdtthltvp on 05-09-2016 - 10:18 trong Bất đẳng thức - Cực trị

Bài 57 (Iran MO 3rd Round Finals). Cho ba số thực dương $a,b,c$ thỏa mãn điều kiện $abc=1.$ Chứng minh rằng

$$\frac {a+b}{(a+b+1)^2}+\frac {b+c}{(b+c+1)^2}+\frac {c+a}{(c+a+1)^2} \geqslant \frac {2}{a+b+c}.$$

 

Áp dụng BĐT $Cauchy-schwarz,$ ta có:

$$(a+b+c)(a+b+\frac{1}{c})\geq (a+b+1)^2$$

Kết hợp với $abc=1$

$$\sum \frac{a+b}{(a+b+1)^2}\geq \sum \frac{a+b}{(a+b+c)(a+b+ab)}$$

Cuối cùng ta chỉ cần chứng minh:

$$\sum \frac{a+b}{a+b+ab}\geq 2\Leftrightarrow \sum \frac{ab}{a+b+ab}\leq 1$$

Đặt $a=\frac{1}{x^3};b=\frac{1}{y^3},c=\frac{1}{z^3}$ thì $xyz=1$ và:

$$\sum \frac{ab}{a+b+ab}=\sum \frac{1}{\frac{1}{a}+\frac{1}{b}+1}=\sum \frac{1}{x^3+y^3+xyz}\leq \sum \frac{1}{xy(x+y)+xyz}=\sum \frac{z}{xyz(x+y+z)}=\sum \frac{z}{x+y+z}=1.$$

Do đó ta có đpcm.




#652352 CMR: AI, DG, BH đồng quy.

Đã gửi bởi tpdtthltvp on 02-09-2016 - 08:17 trong Hình học

Cho hình bình hành ABCD. Qua điểm I trong hình bình hành vẽ đường thẳng song song với AB cắt AD, BC lần lượt tại E, G. Qua I vẽ đường thẳng song song với AD cắt AB, DC lần lượt tại F, H. CMR: AI, DG, BH đồng quy.

TH.JPG

 

Gọi giao điểm của $AI$ và $BH$ là $K.$ Ta sẽ chứng minh $D,K,G$ thẳng hàng.

Áp dụng định lí Menelaus vào tam giác $HBF$ có $A,I,K$ thằng hàng ta được:

$$\frac{AF}{AB}.\frac{IH}{IF}.\frac{KB}{KH}=1\Leftrightarrow \frac{KB}{KH}=\frac{AB}{AF}.\frac{IF}{IH}$$

Do đó tam giác $BHC$ có: 

$$\frac{DH}{DC}.\frac{KB}{KH}.\frac{GC}{GB}=\frac{DH}{DC}.\frac{GC}{GB}.\frac{AB}{AF}.\frac{IF}{IH}=(\frac{DH}{DC}.\frac{AB}{AF}).(\frac{GC}{GB}.\frac{IF}{IH})=1$$

Theo định lí Menelaus suy ra $D,K,G$ thẳng hàng. 

Ta có đpcm.




#651999 Tìm $a,b,p$ thỏa mãn: $2^a+p^b=19^a.$

Đã gửi bởi tpdtthltvp on 30-08-2016 - 20:39 trong Số học

Bài toán $(Italy \;\;\ TST  \;\;\ 2003):$ Tìm bộ số nguyên $(a,b,p)$ sao cho $a,b$ là số nguyên dương và $p$ là số nguyên tố thỏa mãn:

$$2^a+p^b=19^a.$$ 




#651254 Cho x,y,z là các số thực dương CMR $(xy+yz+zx)[\frac{1}...

Đã gửi bởi tpdtthltvp on 25-08-2016 - 20:32 trong Bất đẳng thức và cực trị

Đây là BĐT $Iran \;\;\;\ 96.$ Bạn có thể xem ở link sau $(Pro.5):$   https://truongvoki9x...uongvoki_bn.pdf




#650949 Dựng điểm $M$ trong tam giác sao cho $MA+MB+MC$ đạt giá t...

Đã gửi bởi tpdtthltvp on 23-08-2016 - 17:08 trong Hình học

cho $\Delta$ABC nhọn, AB<AC. Hãy dựng điểm M nằm trong $\Delta$ sao cho:

MA+MB+MC đạt giá trị nhỏ nhất

MAMBMC.JPG

 Dựng các tam giác đều $AMN,ACP($như hình vẽ$).$ Dễ dàng chứng minh được: $\Delta AMC=\Delta ANP(c.g.c)$ suy ra $MC=NP.$

 Do đó: $MA+MB+MC=MB+MN+NP\geq BP$ không đổi. Dấu $"="$ xảy ra khi $B,M,N,P$ thằng hàng $\Leftrightarrow \widehat{AMB}=\widehat{AMC}=\widehat{BMP}=120^{\circ}$




#650913 Topic Ôn thi HSG 9 2015-2016 (Hình học)

Đã gửi bởi tpdtthltvp on 23-08-2016 - 12:16 trong Chuyên đề toán THCS

Cho tam giác $ABC$ có $H$ là trực tâm. Chứng minh: $\frac{HA}{BC}+\frac{HB}{CA}+\frac{HC}{AB}\geq \sqrt{3}$

3.JPG

 Gọi các đường cao của tam giác là $AM,BN,CP.$

 Ta có:  $\Delta APH\sim \Delta AMB(g.g)\Rightarrow \frac{AH}{AB}=\frac{AP}{AM}$

 Do đó: $\frac{HA}{BC}.\frac{HC}{AB}=\frac{HA}{AB}.\frac{HC}{BC}=\frac{AP}{AM}.\frac{HC}{BC}=\frac{\frac{1}{2}.AP.HC}{\frac{1}{2}.AM.BC}=\frac{S_{AHC}}{S_{ABC}}$

 Chứng minh hoàn toàn tương tự ta cũng được: $\frac{HA.HB}{AC.BC}=\frac{S_{AHB}}{S_{ABC}}; \;\;\;\ \frac{HB.HC}{AB.AC}=\frac{S_{HBC}}{S_{ABC}}$

 Suy ra: $\frac{HA}{BC}.\frac{HC}{AB}+\frac{HA.HB}{AC.BC}+\frac{HB.HC}{AB.AC}=\frac{S_{AHC}+S_{AHB}+S_{BHC}}{S_{ABC}}=1$

 Mà ta có BĐT quen thuộc sau: $(a+b+c)^2 \geq 3(ab+bc+ca)$. Áp dụng vào được:

$$(\frac{HA}{BC}+\frac{HB}{CA}+\frac{HC}{AB})^2\geq 3(\frac{HA}{BC}.\frac{HC}{AB}+\frac{HA.HB}{AC.BC}+\frac{HB.HC}{AB.AC})=3\Leftrightarrow \frac{HA}{BC}+\frac{HB}{CA}+\frac{HC}{AB}\geq \sqrt{3}(\text{đpcm}).$$




#650752 13 tóc đỏ, 15 tóc vàng, 17 tóc xanh. 2 có màu tóc khác nhau gặp nhau, tóc của...

Đã gửi bởi tpdtthltvp on 22-08-2016 - 09:44 trong Tổ hợp và rời rạc

Ở Vương quốc "Sắc màu kì ảo'' có $45$ hiệp sĩ: $13$ hiệp sĩ tóc đỏ, $15$ hiệp sĩ tóc vàng, $17$ hiệp sĩ tóc xanh. Khi hai hiệp sĩ có màu tóc khác nhau gặp nhau, tóc của họ sẽ lập tức đổi sang màu thứ ba. Hỏi có thể có một lúc nào đó, tất cả hiệp sĩ đều có màu tóc giống nhau?

 

 Ta thấy: số các hiệp sĩ của mỗi màu tóc khác nhau khi chia cho $3$ sẽ được cả $3$ số dư $(13\equiv 1;15\equiv 0;17\equiv 2(\mod 3))$

 Nên sau mỗi lần gặp nhau, số dư của số các hiệp sĩ có màu tóc khác nhau khi chia cho $3$ vẫn có đầy đủ các số dư $0;1;2$ (Ví dụ đầu tiên hiệp sĩ tóc đỏ gặp hiệp sĩ tóc vàng thì số hiệp sĩ tóc đỏ là $12\equiv 0(\mod3)$, số hiệp sĩ tóc vàng là $14\equiv 2(\mod3)$ và số hiệp sĩ tóc xanh là $19\equiv 1( \mod3)$

 Giả sử một lúc nào đó, tất cả các hiệp sĩ có màu tóc giống nhau thì số dư khi chia cho $3$ của cả $3$ loại cùng là $0$ $(45\equiv 0\equiv 0\;\;\;\;\ (\mod3))$ (Vô lí).

 Vậy tất cả các hiệp sĩ không thể có màu tóc giống nhau ở mọi thời điểm. 

 




#650745 Chứng minh rằng những điểm này có thể phân thành $n$ cặp sao cho cá...

Đã gửi bởi tpdtthltvp on 22-08-2016 - 08:03 trong Toán rời rạc

Cho $2n$ điểm trên mặt phẳng, trong đó không có $3$ điểm nào thẳng hàng. Chứng minh rằng những điểm này có thể phân thành $n$ cặp sao cho các đoạn thẳng nối chúng không cắt nhau.



#650372 Tính dao động tần số của con lắc lò xo.

Đã gửi bởi tpdtthltvp on 19-08-2016 - 18:06 trong Các môn tự nhiên (Vật lý, Hóa học, Sinh học, Công nghệ)

Gọi $M,N,I$ là các điểm trên một lò xo nhẹ, được treo thẳng đứng ở trên điểm $O$ cố định. Khi lò xo có chiều dài tự nhiên thì $OM=MN=NI=10 cm.$ Găn vật nhỏ vào đầu dưới $I$ của lò xo và kích thích để vật dao động điều hòa theo phương thẳng đứng. Trong quá trình dao động, tỉ số độ lớn lực kéo lớn nhất và độ lớn lực kéo nhỏ nhất tác dụng lên $O$ bằng $3,$ lò xo dãn đều, khoảng cách lớn nhất giữa $2$ điểm $M$ và $N$ là $12 cm$ lấy $\pi ^2=10.$ Vật dao động với tần số là:

$A.2,9 Hz \;\;\;\;\;\;\;\;\;\;\ B.2,5Hz \;\;\;\;\;\;\;\;\;\;\ C.3,5 Hz \;\;\;\;\;\;\;\;\;\;\ 1,7 Hz.$




#650368 cho mình hỏi cách xem bài viết của một thành viên cụ thể k

Đã gửi bởi tpdtthltvp on 19-08-2016 - 17:34 trong Hướng dẫn - Trợ giúp - Giải đáp thắc mắc khi sử dụng Diễn đàn

cho mình hỏi cách xem bài viết của một thành viên cụ thể k :icon13:  :icon13:  :icon13:  :icon13:  :icon13:  :icon13:  :icon13:  :icon13:

Bạn vào trang cá nhân của thành viên đó rồi ấn vào Tìm kiếm

tik.PNG

Nếu muốn xem riêng chủ đề hay bài viết thì nhấn vào Chỉ chủ đề hoặc Chỉ bài viết.




#650024 Chứng minh A là số hữu tỉ

Đã gửi bởi tpdtthltvp on 17-08-2016 - 11:41 trong Đại số

Bài 1: Biết a,b,c,d $\epsilon \mathbb{Q}$, a+b+c+d=0. Chứng minh $\sqrt{(ab-cd)(bc-da)(ca-bd)}$ là số hữu tỉ

 

Làm nốt :D

Từ giả thiết ta có $-c=a+b+d$ suy ra $-cd=ad+bd+d^2$

Do đó: $ab-cd=ab+ad+bd+d^2=(a+d)(b+d)$

Tương tự ta cũng được: $bc-da=(b+d)(c+d)$ và $ca-bd=(c+d)(a+d)$

Vậy: $\sqrt{(ab-cd)(bc-da)(ca-bd)}=\sqrt{[(a+d)(b+d)(c+d)]^2}=\left | (a+d)(b+d)(c+d) \right |$ là số hữu tỉ.




#649862 Topic: [LTDH] Mỗi ngày hai bất đẳng thức.

Đã gửi bởi tpdtthltvp on 16-08-2016 - 11:56 trong Bất đẳng thức và cực trị

Bài 20: Cho ba số thực dương $a,b,c$ thỏa mãn: $28(\frac{1}{a^2}+\frac{1}{b^2}+\frac{1}{c^2})=4(\frac{1}{ab}+\frac{1}{bc}+\frac{1}{ca})+2013$.

Tìm GTLN của:

$P=\frac{1}{\sqrt{5a^2+2ab+b^2}}+\frac{1}{\sqrt{5b^2+2bc+c^2}}+\frac{1}{\sqrt{5c^2+2ca+a^2}}$

Bài 20:

Từ giả thiết suy ra:

$$28(\frac{1}{a}+\frac{1}{b}+\frac{1}{c})^2=60(\frac{1}{ab}+\frac{1}{bc}+\frac{1}{ca})+2013\leq 20(\frac{1}{a}+\frac{1}{b}+\frac{1}{c})^2+2013$$

$$\Leftrightarrow 8(\frac{1}{a}+\frac{1}{b}+\frac{1}{c})^2\leq 2013\Leftrightarrow \frac{1}{a}+\frac{1}{b}+\frac{1}{c}\leq \sqrt{\frac{2013}{8}}$$

Mặt khác ta có:

$$P=\sum \frac{1}{\sqrt{5a^2+2ab+b^2}}=\sum \frac{1}{\sqrt{(2a)^2+(a+b)^2}}\leq \sum \frac{1}{\sqrt{\frac{(3a+b)^2}{2}}}=\sqrt{2}\sum \frac{1}{3a+b}$$

Do đó:

$$P\leq \sqrt{2}\sum \left [ \frac{1}{16}(\frac{3}{a}+\frac{1}{b}) \right ]=\frac{\sqrt{2}}{4}(\frac{1}{a}+\frac{1}{b}+\frac{1}{c})\leq \frac{\sqrt{2}}{4}.\sqrt{\frac{2013}{8}}=\sqrt{\frac{2013}{64}}$$

Vậy $\max P=\sqrt{\frac{2013}{64}}.$ Dấu đẳng thức xảy ra khi $a=b=c=2\sqrt{\frac{6}{671}}$




#649528 Topic: [LTDH] Mỗi ngày hai bất đẳng thức.

Đã gửi bởi tpdtthltvp on 14-08-2016 - 07:43 trong Bất đẳng thức và cực trị

Bài 15: Cho $a,b,c>0$. Tìm giá trị nhỏ nhất của biểu thức:$P=\frac{4a^3+3b^3+2c^3-3b^2c}{(a+b+c)^3}$

 

Áp dụng bất đẳng thức $AM-GM:$

$$2b^3+c^3=b^3+b^3+c^3\geq 3b^2c$$

Do đó:

$$P\geq \frac{4a^3+b^3+c^3}{(a+b+c)^3}$$

Theo BĐT $Holder,$ ta có:

$$(4a^3+b^3+c^3)(\frac{1}{2}+1+1)(\frac{1}{2}+1+1)\geq (a+b+c)^3\Leftrightarrow \frac{4a^3+b^3+c^3}{(a+b+c)^3}\geq \frac{4}{25}$$

Vậy $\min P=\frac{4}{25}\Leftrightarrow a=\frac{1}{2}b=\frac{1}{2}c$




#649376 Cho $a^2+b^2\vdots ab$. Tính: $A=\frac{a^2+b^2...

Đã gửi bởi tpdtthltvp on 13-08-2016 - 12:07 trong Đại số

Bạn giải thích chỗ này được ko.  Do tính nhỏ nhất của $a_0+b_0$ nên $t \geq a_0.$ 

Bạn giả sử $a_0 \geq b_0.$.  Theo mình thấy thì:

$a^2_0\geq b^2_0=t.a_0$

$\Rightarrow a_0\geq t$

Dễ dàng suy ra được $t$ cũng nguyên dương. Các cặp $(a_0;b_0);(b_0;t)$ đều thỏa mãn phương trình bậc $2$ đang xét mà $a_0+b_0$ nhỏ nhất nên $a_0+b_0 \leq b_0+t.$ Do đó $a_0\leq t$




#649239 Cho $a^2+b^2\vdots ab$. Tính: $A=\frac{a^2+b^2...

Đã gửi bởi tpdtthltvp on 12-08-2016 - 20:05 trong Đại số

Cho $a, b$ là các số nguyên dương thõa mãn: $a^2+b^2\vdots ab$

Tính giá trị của biểu thức: $A=\frac{a^2+b^2}{2ab}$

Giải chi tiết nhé!!

Đặt $k=\frac{a^2+b^2}{ab}$, cố định $k$ và trong các cặp $(a,b)$ nguyên dương thỏa mãn $k$ nguyên, chọn ra cặp $(a_0,b_0)$ thỏa mãn $a_0+b_0$ nhỏ nhất và $a_0 \geq b_0.$

Xét phương trình bậc $2$ ẩn $x:$    $x^2-kb_0x+b_0^2=0.$

Dễ thấy phương tình này có $1$ nghiệm là $a_0,$ gọi nghiệm còn lại là $t$ thì theo định lí $Viete,$ ta có: $\left\{\begin{matrix} t+a_0=kb_0 \;\;\ (1) \\ ta_0=b_0^2 \end{matrix}\right.$

Do tính nhỏ nhất của $a_0+b_0$ nên $t \geq a_0.$ Từ $(1)$ suy ra $kb_0\geq a_0+a_0=2a_0\Rightarrow \frac{a_0}{b_0}\leq \frac{k}{2}$

Như vậy: $k=\frac{a_0}{b_0}+\frac{b_0}{a_0}\leq \frac{k}{2}+1\Rightarrow k\leq 2$

Do đó, $k\in \left \{ 1;2 \right \}.$ mà $a^2+b^2\geq 2ab>ab$ nên $k>1.$ Từ đó $k=2$ suy ra $a=b.$ rồi tính được $A=1$

Vậy $A=1$




#648552 CMR: $\frac{AN}{NB} + \frac{AM}...

Đã gửi bởi tpdtthltvp on 08-08-2016 - 12:22 trong Hình học

Cho $\Delta ABC ; O \in \Delta ABC$; AO cắt BC tại D; BO cắt AC tại M; CO cắt AB tại N. CMR: $\frac{AN}{NB} + \frac{AM}{MC} = \frac{AO}{OD}$

 

Van Oben.PNG

Đây là định lí Van Oben mà!

Chứng minh:

Qua $A$ kẻ đường thẳng song song với $BC$ cắt $BM$ tại $P$ và $CN$ tại $Q.$

Theo định lí $Thales,$ ta có:

$$\frac{AO}{OD}=\frac{AP}{BD}=\frac{AQ}{CD}=\frac{AP+AQ}{BD+CD}=\frac{AP}{BC}+\frac{AQ}{BC}=\frac{AM}{MC}+\frac{AN}{NC}\;\;\;\;\;\ (\text{đpcm}).$$




#648248 $HF=KE$

Đã gửi bởi tpdtthltvp on 06-08-2016 - 18:05 trong Hình học

Cho tam giác nhọn $ABC$, đường cao $BE$ và $CF$. $BH$ và $CK$ vuông góc với $EF$.

Chứng minh rằng: $HF=KE$

Ta có: $\widehat{BFC}=\widehat{BEC}=90^{\circ}$ suy ra tứ giác $BFEC$ nội tiếp đường tròn $(O)$ với $O$ là trung điểm của $BC.$

Kẻ $OI$ vuông góc với $HK$ tại $I$ thì $OI$ là đường trung bình hình thang $BHKC$ nên $I$ là trung điểm của $HK. \;\;\;\;\;\;\;\ (1)$

Mà $OI$ vuông góc với dây $EF$ của $(O)$ nên $I$ là trung điểm của $EF \;\;\;\;\;\;\;\;\ (2)$

Từ $(1),(2)$ suy ra $HF=EK$ (đpcm).

 

 

NOITIEP.PNG




#648171 Chứng minh: $\sum\frac{3a+b}{2a+c}\ge...

Đã gửi bởi tpdtthltvp on 06-08-2016 - 08:14 trong Bất đẳng thức và cực trị

Bài 3: Cho a,b,c dương. Chứng minh:

$\frac{a-b}{a+2b+c}+\frac{b-c}{b+2c+d}+\frac{c-d}{c+2d+a}+\frac{d-a}{d+2a+b}\geq 0$

 

Bất đẳng thức cần chứng minh tương đương:

$$\sum \frac{2a-2b}{a+2b+c}\geq 0\Leftrightarrow \sum \frac{3a+c}{a+2b+c}\geq 3$$

Áp dụng $Cauchy-schwarz:$

$$\sum \frac{3a+c}{a+2b+c}=\sum \frac{(3a+c)^2}{(3a+c)(a+2b+c)}\geq \frac{16(a+b+c)^2}{\sum (3a+c)(a+2b+c)}=\frac{16(a^2+b^2+c^2)+32(ab+bc+ca)}{4(a^2+b^2+c^2)+12(ab+bc+ca)}$$

Cần chứng minh:

$$\frac{16(a^2+b^2+c^2)+32(ab+bc+ca)}{4(a^2+b^2+c^2)+12(ab+bc+ca)}\geq 3\Leftrightarrow 4(a^2+b^2+c^2)\geq 4(ab+bc+ca)\Leftrightarrow a^2+b^2+c^2\geq ab+bc+ca,\text{ đúng}$$

Suy ra đpcm. Dấu $"="$ xảy ra khi $a=b=c.$




#647672 Hỏi mỗi người đi hết quãng đường AB (.) thời gian bao lâu

Đã gửi bởi tpdtthltvp on 02-08-2016 - 20:26 trong Đại số

Tìm quan hệ giữa a,b,c để phương trình:

$(x+a)^4+(x+b)^4=c$ có nghiệm.

Từ giả thiết suy ra $c\geq 0.$

Đặt $z=x+\frac{a+b}{2}, m=\frac{a-b}{2}$ thì giả thiết viết lại thành $(z-m)^4+(z+m)^4=c\Leftrightarrow 2z^4+12m^2z^2+2m^4-c=0 \;\;\;\;\;\;\;\;\ (1)$

Lại đặt $z^2=y \;\;\ (y\geq 0)$ thì $(1)\Leftrightarrow 2y^2+12m^2y+2m^4-c=0. \;\;\;\;\ (2)$

Cần tìm mối quan hệ giữa $m,c$ để $(2)$ có nghiệm không âm.

ĐK để cả $2$ nghiệm của $(2)$ đều âm là: $\left\{\begin{matrix} \Delta '=32m^4+2c\geq 0 \\ S=x_1+x_2=-6m^2<0 \\ P=x_1x_2=\frac{2m^4-c}{2}>0 \end{matrix}\right.\Leftrightarrow  2m^4>c $ (Vì $c\geq 0$)

Vậy để phương trình có nghiệm thì: $2m^2\leq c\Leftrightarrow (a-b)^4\leq 8c$




#647586 $\sqrt{10}\left [ \left ( 1+\sqrt{10...

Đã gửi bởi tpdtthltvp on 02-08-2016 - 06:44 trong Số học

   CM pt sau nguyên

 

$\sqrt{10}\left [ \left ( 1+\sqrt{10} \right )^{100} -\left ( 1-\sqrt{10} \right )^{100}\right ]$

 

Ps : Cho minh xin công thức chung ý.... Cách bt ra rồi

 Công thức chung: Với mọi số nguyên không âm $m,n$ và $k$ nguyên thì biểu thức sau nguyên:

$$\sqrt{m}\left [ (k+\sqrt{m})^n-(k-\sqrt{m})^n \right ]$$

 

Chứng minh:

Theo công thức khai triển $(x+y)^n$ ta có:

$(k+\sqrt{m})^n=A+B\sqrt{2}$      và     $(k-\sqrt{m})^n=A-B\sqrt{2}$ (Với $A,B\in \mathbb{N}$)

Do đó: $\sqrt{m}\left [ (k+\sqrt{m})^n-(k-\sqrt{m})^n \right ]=\sqrt{m}(A+B\sqrt{m}-A+B\sqrt{m})=\sqrt{m}.2B\sqrt{m}=2mB.$

Suy ra điều phải chứng minh.




#647563 Tìm số $\overline{a_{1}a_{2}...a_{n-1...

Đã gửi bởi tpdtthltvp on 01-08-2016 - 22:11 trong Số học

Bài toán. Tìm số $\overline{a_{1}a_{2}...a_{n-1}a_{n}}$ thỏa mãn tính chất $\sqrt{\overline{a_{1}a_{2}...a_{n-1}a_{n}}}=a_{1}+a_{2}+...+a_{n-1}+a_{n}$.

Giả thiết tương đương: $\overline{a_{1}a_{2}...a_{n-1}a_{n}}=(a_{1}+a_{2}+...+a_{n-1}+a_n)^2$

Suy ra: $\overline{a_{1}a_{2}...a_{n-1}a_{n}}\leq (9n)^2=81n^2.$ Ta sẽ chứng minh với $n\geq 5$ thì $\overline{a_{1}a_{2}...a_{n-1}a_{n}}> (9n)^2\;\;\;\;\;\;\;\(1)$

Thật vậy, với $n=5$ thì $(1)$ đúng. Giả sử đúng với $n=k(k\geq 5),$ thì $\overline{a_{1}a_{2}...a_{n-1}a_{k}}> 81k^2.$Ta đi chứng minh đúng với $n=k+1.$ Với $n=k+1,$ ta có: $(1)\Leftrightarrow \overline{a_{1}a_{2}...a_{n-1}a_{k+1}}> 81(k+1)^2\Leftrightarrow 10.\overline{a_{1}a_{2}...a_{n-1}a_{n}}+a_{k+1}>81k^2+162k+81$

Mà $\overline{a_{1}a_{2}...a_{n-1}a_{n}}>81k^2\Rightarrow 10.\overline{a_{1}a_{2}...a_{n-1}a_{n}}+a_{k+1}>810k^2.$ Ta thấy $810k^2>81k^2+162k+81\Leftrightarrow 719k^2-162k-81>0.$ Đặt $f(x)=719k^2-162k-81$ thì $f(x)$ đồng biến trên $\left [ 5;\infty+  \right ]$ nên $f(x)\geq f(5)>0.$

Từ đó suy ra $n\leq 4\Rightarrow n\in \left \{ 1;2;3;4 \right \}.$

  • $n=1:$ thì $a_1=a_1^2\Rightarrow a_1=1.$ Số cần tìm là $1$
  • $n=2:$ thì $\overline{a_1a_2}=(a_1+a_2)^2.$ Để ý $a_2\in \left \{ 0;1;4;5;6;9 \right \}.$ Thế vào phương trình $10a_1+a_2=(a_1+a_2)^2$ được số cần tìm là $81$
  • $n=3:$ thì $\overline{a_1a_2a_3}=(a_1+a_2+a_3)^2\Rightarrow 10\leq a_1+a_2+a_3\leq 27.$ Thử từng TH thấy không có số nào thỏa mãn :D
    P/S
  • $n=4$ thì $\overline{a_1a_2a_3a_4}=(a_1+a_2+a_3+a_4)^2\Rightarrow 31\leq a_1+a_2+a_3+a_4\leq 36.$ Cái này ít TH nên thử được, cũng vô nghiệm.

Vậy các số cần tìm là $1;81$